Violympic toán 7

Tuấn Đinh

-1-\(\dfrac{1}{2}-\dfrac{1}{4}-\dfrac{1}{8}-...-\dfrac{1}{1024}\)

Nguyễn Thanh Hằng
6 tháng 9 2017 lúc 20:17

Đặt :

\(H=-1-\dfrac{1}{2}-\dfrac{1}{4}-\dfrac{1}{8}-..........-\dfrac{1}{1024}\)

\(\Leftrightarrow H=-1-\left(\dfrac{1}{2}+\dfrac{1}{4}+...........+\dfrac{1}{1024}\right)\)

Đặt :

\(T=\dfrac{1}{2}+\dfrac{1}{4}+.......+\dfrac{1}{1024}\)

\(\Leftrightarrow T=\dfrac{1}{2}+\dfrac{1}{2^2}+..........+\dfrac{1}{2^{10}}\)

\(\Leftrightarrow2T=1+\dfrac{1}{2}+\dfrac{1}{2^2}+.........+\dfrac{1}{2^9}\)

\(\Leftrightarrow2T-T=\left(1+\dfrac{1}{2}+.....+\dfrac{1}{2^9}\right)-\left(\dfrac{1}{2}+\dfrac{1}{2^2}+.....+\dfrac{1}{2^{10}}\right)\)

\(\Leftrightarrow T=1-\dfrac{1}{2^{10}}\)

\(\Leftrightarrow H=-1-\left(1-\dfrac{1}{2^{10}}\right)\)

\(\Leftrightarrow H=-1-1+\dfrac{1}{2^{10}}\)

\(\Leftrightarrow H=-2+\dfrac{1}{2^{10}}\)

Bình luận (0)
Phương Trâm
6 tháng 9 2017 lúc 20:18

Đặt \(A=-1-\dfrac{1}{2}-\dfrac{1}{4}-\dfrac{1}{8}-...-\dfrac{1}{1024}\)

\(A=-\left(\dfrac{1}{2}+\dfrac{1}{4}+\dfrac{1}{8}+...+\dfrac{1}{1024}\right)\)

Đặt \(B=\dfrac{1}{2}+\dfrac{1}{4}+\dfrac{1}{8}+...+\dfrac{1}{1024}\)

\(2B=1+\dfrac{1}{2}+\dfrac{1}{4}+\dfrac{1}{8}+...+\dfrac{1}{512}\)

\(2B-B=1-\dfrac{1}{1024}\)

\(\Rightarrow B=\dfrac{1023}{1024}\)

\(\Rightarrow A=-\dfrac{1023}{1024}\)

Bình luận (1)
 Mashiro Shiina
7 tháng 9 2017 lúc 13:51

Đặt

\(W=-1-\dfrac{1}{2}-\dfrac{1}{4}-\dfrac{1}{8}-....-\dfrac{1}{1024}\)

\(W=-1-\left(\dfrac{1}{2}+\dfrac{1}{4}+\dfrac{1}{8}+....+\dfrac{1}{1024}\right)\)

\(W=-1-\left(\dfrac{1}{2^1}+\dfrac{1}{2^2}+\dfrac{1}{2^3}+....+\dfrac{1}{2^{10}}\right)\)

Đặt:

\(A=\dfrac{1}{2^1}+\dfrac{1}{2^2}+\dfrac{1}{2^3}+....+\dfrac{1}{2^{10}}\)

\(2A=2\left(\dfrac{1}{2^1}+\dfrac{1}{2^2}+\dfrac{1}{2^3}+....+\dfrac{1}{2^{10}}\right)\)

\(2A=1+\dfrac{1}{2^1}+\dfrac{1}{2^2}+...+\dfrac{1}{2^9}\)

\(2A-A=\left(1+\dfrac{1}{2^1}+\dfrac{1}{2^2}+...+\dfrac{1}{2^9}\right)-\left(\dfrac{1}{2^1}+\dfrac{1}{2^2}+\dfrac{1}{2^3}+....+\dfrac{1}{2^{10}}\right)\)

\(A=1-\dfrac{1}{2^{10}}\)

Thay \(A\) vào \(W\) ta có:

\(W=-1-1+\dfrac{1}{2^{10}}=-2+\dfrac{1}{2^{10}}\)

Bình luận (0)

Các câu hỏi tương tự
Nguyễn Thị Giang Thanh
Xem chi tiết
NGUYỄN THỊ THANH ĐÀO
Xem chi tiết
Ichigo
Xem chi tiết
Hoàng Gia Hân
Xem chi tiết
thanh nguyen van long
Xem chi tiết
Nguyễn Anh Thư
Xem chi tiết
za hân
Xem chi tiết
Hoa Nguyễn Lệ
Xem chi tiết
Xem chi tiết